LSAT and Law School Admissions Forum

Get expert LSAT preparation and law school admissions advice from PowerScore Test Preparation.

User avatar
 Dave Killoran
PowerScore Staff
  • PowerScore Staff
  • Posts: 5852
  • Joined: Mar 25, 2011
|
#27111
Complete Question Explanation
(The complete setup for this game can be found here: lsat/viewtopic.php?t=49)

The correct answer choice is (C)

If the stand does not carry W, then by the contrapositive of the third rule the stand cannot carry O. The only remaining fruits that could be carried by the stand are then F, T, K, and P. However, from the first rule K and P cannot be carried at the same time, and the only fruits that can then be carried are F, T, and the choice of K or P, or a maximum of three fruits. Answer choice (C) is therefore correct.

Answer choice (B) is incorrect because although the stand can carry two kinds of fruit, it does not have to carry at least two kinds of fruit (it could carry only one kind of fruit, K or T).

Answer choices (D) and (E) are incorrect because although the stand cannot carry O, it could carry P or K.
User avatar
 LSATQueen2024
  • Posts: 9
  • Joined: Jan 16, 2024
|
#105479
I'm a bit confused about the third rule and its application in this context. If the stand doesn't carry watermelons, why can't we place pears and oranges? Could you explain why oranges couldn't be in the stand? Is the rule implying that if both pears and watermelons are absent, then oranges won't be on the stand? Is that why watermelons aren't available on the stand?"

#Help!
User avatar
 Jeff Wren
PowerScore Staff
  • PowerScore Staff
  • Posts: 389
  • Joined: Oct 19, 2022
|
#105497
Hi LSATQueen,

The third rule states that if the stand carries oranges, then it carries pears and watermelons.

The could be diagrammed:

O -> P + W

Now the tricky part comes in taking the contrapositive, which I recommend diagramming out.

When taking the contrapositive of a conditional statement with multiple sufficient or necessary conditions, there is a special rule to remember. You still reverse the terms and negate the terms, but the word "and" (which we show using the + symbol in the diagram) gets changed to the word "or."

So the contrapositive of rule 3 would be:

If not Pears or Not Watermelons, then not Oranges.

Which could be diagrammed

not P or not W -> not O

(normally I'd use slashes instead of writing the word "not")

Given this contrapositive, if we know that the stand does not carry Watermelons (as specified in question 4), that is enough by itself to tell us that the stand does not carry Oranges.

This doesn't tell us anything about Pears; however, Pears could still be in or out at this point.

Diagramming multiple sufficient and necessary conditions is discussed in lesson 2 of The PowerScore LSAT Course and in chapter 6 of "The Logical Reasoning Bible."

Get the most out of your LSAT Prep Plus subscription.

Analyze and track your performance with our Testing and Analytics Package.